Đến nội dung

Messi10597 nội dung

Có 398 mục bởi Messi10597 (Tìm giới hạn từ 16-05-2020)



Sắp theo                Sắp xếp  

#492324 $\boxed{\text{Chuyên Đề}}$ Phương trình vô tỉ - Hệ phương...

Đã gửi bởi Messi10597 on 11-04-2014 - 22:50 trong Phương trình, hệ phương trình và bất phương trình

143) $\left\{\begin{matrix}2x^2+3y^2-4xy=3 & & \\ 2x^2-y^2=7 & & \end{matrix}\right.$

Từ hệ suy ra $7(2x^{2}+3y^{2}-4xy)=3(2x^{2}-y^{2})$

                     $\Leftrightarrow 8x^{2}-28xy+24y^{2}=0$

                     $\Leftrightarrow (2x-3y)(x-2y)=0$

Đến đây thế vào PT dưới là xong




#492452 $\boxed{\text{Chuyên Đề}}$ Phương trình vô tỉ - Hệ phương...

Đã gửi bởi Messi10597 on 12-04-2014 - 16:48 trong Phương trình, hệ phương trình và bất phương trình

154) $\left\{\begin{matrix}x^3-9y^2+27y-27=0 & & \\ y^3-9z^2+27z-27=0 & & \\ z^3-9x^2+27x-27=0 \end{matrix}\right.$

Hệ $\Leftrightarrow \left\{\begin{matrix} x^{3} =9y^{2}-27y+27=f(y)& & & \\ y^{3} =9z^{2}-27z+27=f(z)& & & \\ z^{3}=9x^{2} -27x+27=f(x)& & & \end{matrix}\right.$

Vì $f(t)=9t^{2}-27t+27=9(t-\frac{3}{2})^{2}+\frac{27}{4}\geq \frac{27}{4}\forall t$

$\Rightarrow x^{3},y^{3},z^{3}\geq\frac{27}{4}\Rightarrow x,y,z\geq \frac{3}{\sqrt[3]{4}}$

${f}'(t)=18t-27> 0\forall t\in [\frac{3}{\sqrt[3]{4}};+\infty ]$

$\Rightarrow f$ là hàm đồng biến trên đoạn $[\frac{3}{\sqrt[3]{4}};+\infty ]$

Không mất tính tổng quá giả xử $x\geq y\geq z\Rightarrow f(x)\geq f(y)\geq f(z)\Rightarrow z^{3}\geq x^{3}\geq y^{3}\Rightarrow z\geq x\geq y$

$\Rightarrow x=y=z$

Ta có: $x^{3}-9x^{2}+27x-27=0$

Đến đây dễ rồi




#508176 Bất đẳng thức chuẩn bị cho kì thi THPTQG 2015-2016

Đã gửi bởi Messi10597 on 21-06-2014 - 10:32 trong Bất đẳng thức và cực trị

Cho a,b,c là các số dương thoả mãn a + b + c = 1. Tìm GTNN của: 

$\frac{a+b}{\sqrt{ab+c}}+\frac{b+c}{\sqrt{bc+a}}+\frac{c+a}{\sqrt{ca+b}}$ 

Ta có: $\sum \frac{a+b}{\sqrt{ab+c}}=\sum \frac{a+b}{\sqrt{(a+c)(b+c)}}\geq \frac{(\sqrt{a+b}+\sqrt{b+c}+\sqrt{c+a})^{2}}{\sqrt{(a+b)(a+c)}+\sqrt{(a+b)(b+c)}+\sqrt{(a+c)(b+c)}}\geq 3$

Dấu bằng xảy ra $a=b=c=\frac{1}{3}$




#582623 Bất đẳng thức chuẩn bị cho kì thi THPTQG 2015-2016

Đã gửi bởi Messi10597 on 17-08-2015 - 16:23 trong Bất đẳng thức và cực trị

Cho $a,b,c>0$ thỏa mãn $a+b+c=1$ .Tìm GTNN của:

$P=\frac{a^{2}}{1+b}+\frac{b^{2}}{1+a}+\frac{4c^{2}}{2+\sqrt{a^{2}+b^{2}}}$




#555134 Topic Tổng Hợp Các Bài Toán Hình Học Phẳng Trong Các Đề Thi Thử THPT Quốc Gia...

Đã gửi bởi Messi10597 on 19-04-2015 - 21:14 trong Phương pháp tọa độ trong mặt phẳng

Một bài nữa :

Câu 18:

Trong mặt phẳng tọa độ Oxy, cho hình thang ABCD vuông tại C, D có BC = 2 AD = 2DC ,
đỉnh C(3;-3) , đỉnh A nằm trên đường thẳng : 3- 2 = 0 , phương trình đường thẳng
DM - 2 = 0 với M là điểm thỏa mãn de-thi-thu-thpt-quoc-gia-mon-toan-nam-20 . Xác định tọa độ các điểm A, D, B ( THPT Hàn Thuyên)

 

Bài này chỉ cần tìm đc M là tìm đc tất các điểm cần tìm

Gọi N là trung điểm BC thì ANCD là hình vuông , M là trung điểm CN

Gọi E là trung điểm AN $\Rightarrow DM\perp CE$ 

$\Rightarrow \overrightarrow{n_{CE}}=\overrightarrow{u_{DM}}=(1;1)\Rightarrow CE:x+y=0$

$H=DM\cap CE\Rightarrow H(1;-1)$

Ta có: $\Delta CHM\sim \Delta DCM\Rightarrow \frac{HM}{CH}=\frac{CM}{DC}=\frac{1}{2}\Rightarrow 2HM=CH$

$M\in MD\Rightarrow M(t;t-2)\Rightarrow 2\sqrt{(t-1)^{2}+(t-2+1)^{2}}=2\sqrt{2}\Leftrightarrow \left | t-1 \right |=1$




#554735 Topic Tổng Hợp Các Bài Toán Hình Học Phẳng Trong Các Đề Thi Thử THPT Quốc Gia...

Đã gửi bởi Messi10597 on 17-04-2015 - 22:31 trong Phương pháp tọa độ trong mặt phẳng

Câu 13: Tự nhiên lục lọi trong topic đề thi thử THPT quốc gia 2015 lại tìm thấy đúng bài cần tìm :lol:

Trong mặt phẳng với hệ trục Oxy, cho tam giác ABC có đường trung tuyến AM và đường cao AH lần lượt có phương trình 13x-6y-2=0,x-2y-14=0. Tìm tọa độ các đỉnh của tam giác ABC biết tâm đường tròn ngoại tiếp của tam giác ABC là I(-6;0). (THPT chuyên Hùng Vương)

Ta tìm đc A(-4;-9)

Gội G là trọng tâm,K là trực tâm tam giác ABC

Dễ dàng cm đc K,G,I thẳng hàng và $\overrightarrow{IG}=\frac{1}{3}\overrightarrow{IK}$ (theo đường thẳng ơle)

khi đó tìm đc điểm K,lại có $\overrightarrow{AK}=2\overrightarrow{IM}$ ,tìm đc M

khi đó ta viếtđc pt BC

Tọa độ B,C là ngiệm của hệ gồm pt BC và pt đường tròn




#566857 Topic Tổng Hợp Các Bài Toán Hình Học Phẳng Trong Các Đề Thi Thử THPT Quốc Gia...

Đã gửi bởi Messi10597 on 19-06-2015 - 14:01 trong Phương pháp tọa độ trong mặt phẳng

Bài 36: Trong mặt phẳng Oxy, tam giác ABC có trực tâm H(5,5), phương trình chứa cạnh cạnh BC là x+y-8=0. Biết đường tròn ngoại tiếp tam giac đi qua 2 điểm M(7,3), N(4,2) tính diện tích ABC.

 

------------

Bạn hãy post bài nghiêm túc hơn nhé!

Mình ko biết vẽ hình đăng lên đâu mn thông cảm nhé 

Gọi giao của AH với đường tròn là K

Ta chứng minh K đối xúng với H qua BC

Ta có $\widehat{KBC}= \widehat{KAC}$ (cùng chắn cung KC)

          $\widehat{KAC}=\widehat{HBC}$ (cùng phụ với $\widehat{ACB}$ )

 Suy ra $\widehat{KBC}=\widehat{HBC}$ ,suy ra tam giác HBK cân tại B,suy ra K đối xúng với H qua BC,từ đó tìm đc K

đến đây dễ rồi




#432067 [TOPIC] Phương trình lượng giác - Các đề thi thử 2012

Đã gửi bởi Messi10597 on 01-07-2013 - 16:32 trong Phương trình, Hệ phương trình Lượng giác

Bài 49. Giải phương trình: $$4\cos x-3\sin x+\dfrac{2}{4\cos x-3\sin x-6}=3$$

Phương trình tương đương với $4cosx-3sinx-6+\frac{2}{4cosx-3sinx-6}+3=0$

Đặt : $t=4cosx-3sinx-6 ;(t\neq 0)$

Phương trình trở thành $t+\frac{2}{t}+3=0\Leftrightarrow t^{2}+3t+2=0$

               $\Leftrightarrow t=-1$   hoặc $t=-2$

Nếu $t=-1\Rightarrow 4cosx-3sinx-6=-1 \Leftrightarrow 4cosx-3sinx-5=0$ (1)

Xét $cos\frac{x}{2}=0\Rightarrow sinx=0;cosx=-1$ không thỏa mãn

Xét $cosx\neq 0$ .Đăt $a=tan\frac{x}{2}$

(1) trở thành $\frac{4(1-a^{2})}{1+a^{2}}-\frac{6a}{1+a^{2}}-5=0$ quy đồng thu đc PT bậc 2

Nếu $t=-2\Rightarrow 4cosx-3sinx-6=-2\Leftrightarrow -3sinx=4(1-cosx)$

                                                                    $\Leftrightarrow -6sin\frac{x}{2}cos\frac{x}{2}=8sin^{2}\frac{x}{2}$

                                                                    $2sin\frac{x}{2}(4sin\frac{x}{2}+3cos\frac{x}{2})=0$

Đến đây chắc dễ rồi,anh chị xem em làm có nhầm chỗ nào không ạ




#432051 [TOPIC] Phương trình lượng giác - Các đề thi thử 2012

Đã gửi bởi Messi10597 on 01-07-2013 - 15:41 trong Phương trình, Hệ phương trình Lượng giác

Mình cũng xin góp một bài :
$\left ( cos2x+cos4x \right )^{2}= 5+cosx$

Em làm thế này ko biết đúng ko

Ta có $cos2x+cos4x\leq 2\Rightarrow (cos2x+cos4x)^{2}\leq 4$

          $5+cosx\geq 4$

Phương trình có nghiệm $\Leftrightarrow (cos2x+cos4x)^{2}=5+cosx=4$ 

                                       $\left\{\begin{matrix} cosx=-1 & & & \\ cos2x=\pm 1 & & & \\ cos4x=\pm 1 & & & \end{matrix}\right.$




#443517 Topic về Lượng giác và vấn đề liên quan

Đã gửi bởi Messi10597 on 17-08-2013 - 09:18 trong Các bài toán Lượng giác khác

2)$8cos^{3}(x+\frac{\pi }{3})=cos3x$

2. Đặt $x+\frac{\pi }{3}=t$ 

PT trở thành: $8cos^{3}t=cos(3t-\pi )=-cos3t$

                      $\Leftrightarrow 8cos^{3}t+4cos^{3}t-3cost=0$

                      $\Leftrightarrow 12cos^{3}t-3cost=0$

                      $\Leftrightarrow cost(2cost-1)(2cost+1)=0$




#505782 Đề thi tuyển sinh lớp 10 ĐHKHTN (2 vòng) năm 2014-2015

Đã gửi bởi Messi10597 on 11-06-2014 - 16:22 trong Tài liệu - Đề thi

Câu hệ:

Hệ$\Leftrightarrow \left\{\begin{matrix} (x-y)(2x+3y)=12 & & \\ (x-y)(xy+6)=12 & & \end{matrix}\right.$

$\Rightarrow 2x+3y=xy+6\Leftrightarrow (x-3)(y-2)=0$




#505790 Đề thi tuyển sinh lớp 10 ĐHKHTN (2 vòng) năm 2014-2015

Đã gửi bởi Messi10597 on 11-06-2014 - 16:47 trong Tài liệu - Đề thi

Thiếu $x=y$ (mà làm rồi mà)

 

Nếu x=y thì có thỏa $(x-y)(xy+6)=12$ đâu




#505601 Đề thi tuyển sinh lớp 10 ĐHKHTN (2 vòng) năm 2014-2015

Đã gửi bởi Messi10597 on 10-06-2014 - 21:11 trong Tài liệu - Đề thi

Câu 4:

Ta có: $1=(ab+bc+ca)^{2}=a^{2}b^{2}+b^{2}c^{2}+c^{2}a^{2}+2abc(a+b+c)$

$\Rightarrow dpcm\Leftrightarrow a^{4}b^{2}+b^{4}c^{2}+c^{4}a^{2}+a^{2}b^{2}+b^{2}c^{2}+c^{2}a^{2}\geq \frac{4}{9}$

$\Leftrightarrow a^{2}b^{2}(a^{2}+1)+b^{2}c^{2}(b^{2}+1)+c^{2}a^{2}(c^{2}+1)\geq \frac{4}{9}$

$\Leftrightarrow a^{2}b^{2}(a+b)(a+c)+b^{2}c^{2}(b+c)(b+a)+c^{2}a^{2}(c+a)(c+b)\geq \frac{4}{9}$

$(a+b)(b+c)(c+a)(\frac{a^{2}b^{2}}{b+c}+\frac{b^{2}c^{2}}{c+a}+\frac{c^{2}a^{2}}{a+b})\geq \frac{4}{9}$

Nhờ biến đổi tương đương ta chứng minh đc $(a+b)(b+c)(c+a)\geq \frac{8}{9}(a+b+c)(ab+bc+ca)=\frac{8}{9}(a+b+c)$

Áp dụng BĐT Cauchy-Schwarz ta có:

$\frac{a^{2}b^{2}}{b+c}+\frac{b^{2}c^{2}}{c+a}+\frac{c^{2}a^{2}}{a+b}\geq \frac{(ab+bc+ca)^{2}}{2(a+b+c)}=\frac{1}{2(a+b+c)}$

vậy ta có đpcm

Dấu bằng xảy ra $\Leftrightarrow a=b=c=\frac{1}{\sqrt{3}}$




#504186 ĐỀ THI TUYỂN SINH THPT CHUYÊN Đại Học Sư Phạm Hà Nội năm 2014

Đã gửi bởi Messi10597 on 05-06-2014 - 13:44 trong Tài liệu - Đề thi

Bầi 3:

1.Xét phương trình hoành độ giao điểm:

$x^{2}=-\frac{2}{3}(m+1)x+\frac{1}{3}\Leftrightarrow 3x^{2}+2(m+1)x-1=0$

${\Delta }'=(m+1)^{2}+3> 0,\forall m\in \mathbb{R}$

suy ra đpcm

2.$f(x_{1})-f(x_{2})=x_{1}[x_{1}^{2}+(m+1)x_{1}-1]-x_{2}[x_{2}^{2}+(m+1)x_{2}-1]$

Do $x_{1};x_{2}$ là nghiệm của PT trên

$\Rightarrow \left\{\begin{matrix} 3x_{1}^{2}+2(m+1)x_{1}-1=0 & & \\ 3x_{2}^{2}+2(m+1)x_{2}-1=0 & & \end{matrix}\right.\Leftrightarrow \left\{\begin{matrix} x_{1}^{2}+(m+1)x_{1}-1=-\frac{1}{2}(x_{1}^{2}+1) & & \\ x_{2}^{2}+(m+1)x_{2}-1=-\frac{1}{2}(x_{2}^{2}+1) & & \end{matrix}\right.$

Và $x_{1}x_{2}=\frac{-1}{3}$

Thay vào ta có 

$f(x_{1})-f(x_{2})=-\frac{1}{2}(x_{1}^{3}+x_{1}-x_{2}-x_{2}^{3})=-\frac{1}{2}[x_{1}^{3}-3x_{1}x_{2}(x_{1}-x_{2})-x_{2}^{3}]=-\frac{1}{2}(x_{1}-x_{2})^{3}$




#463561 Chuyên đề số phức luyện thi Đại Học

Đã gửi bởi Messi10597 on 11-11-2013 - 14:54 trong Tổ hợp - Xác suất và thống kê - Số phức

Bài 30: Tìm số phức z thỏa $\left ( \frac{z+i}{z-i} \right )^{4}=1$

Ta có: $(\frac{z+i}{z-i})^{4}=1\Leftrightarrow (\frac{z+i}{z-i})^{2}=\pm 1$

Giải $(\frac{z+i}{z-i})^{2}=1\Leftrightarrow \frac{z+i}{z-i}=\pm 1\Leftrightarrow z=0$

Giải $(\frac{z+i}{z-i})^{2}=-1\Leftrightarrow (\frac{z+i}{z-i})^{2}=i^{2}\Leftrightarrow (\frac{z+i}{z-i}-i)(\frac{z+i}{z-i}+1)=0\Leftrightarrow z=\pm 1$




#510752 Đề thi khối A, A1

Đã gửi bởi Messi10597 on 04-07-2014 - 16:00 trong Thi TS ĐH

cau9.png

Em lấy ảnh này trên mạng,mọi người tham khảo




#505862 topic các bài toán bất đẳng thức

Đã gửi bởi Messi10597 on 11-06-2014 - 20:47 trong Bất đẳng thức và cực trị

tiếp nhé:

Cho các số thực dương a,b chứng minh rằng:

 

b, $\frac{a^{3}-11b^{3}}{4b^{2}+ab}\geq a-3b$

b. $dpcm\Leftrightarrow a^{3}-11b^{3}\geq (a-3b)(4b^{2}+ab)=4ab^{2}+a^{2}b-12b^{3}-3ab^{2}$

   $\Leftrightarrow a^{3}+b^{3}\geq a^{2}b+ab^{2}=ab(a+b)$ 

BĐT trên đúng do $a^{3}+b^{3}=(a+b)(a^{2}-ab+b^{2})\geq (a+b)(2ab-ab)=ab(a+b)$




#513919 TOPIC: Xoay quanh $\sum \frac{1}{x^2+x+1}...

Đã gửi bởi Messi10597 on 19-07-2014 - 16:06 trong Bất đẳng thức và cực trị

$\bigstar$ VD 11:

 

$\sum \sqrt{\frac{a^2}{a^2+7ab+b^2}}\geq 1$

 

$\blacksquare \blacksquare \blacksquare$

 

 

$dpcm\Leftrightarrow \sum \frac{1}{\sqrt{1+7\frac{b}{a}+\frac{b^{2}}{a^{2}}}}\geq 1$

Đặt : $\sqrt{\frac{b}{a}}=x;\sqrt{\frac{c}{b}}=y;\sqrt{\frac{a}{c}}=z\Rightarrow xyz=1$

BĐT trở thành $\sum \frac{1}{\sqrt{x^{4}+7x^{2}+1}}\geq 1$

Ta chứng minh $\frac{1}{\sqrt{x^{4}+7x^{2}+1}}\geq \frac{1}{x^{2}+x+1}$ (1)

Thật vậy : 

      $(1)\Leftrightarrow (x^{2}+x+1)^{2}-(x^{4}+7x^{2}+1)\geq 0\Leftrightarrow 2x(x-1)^{2}\geq 0$ (luôn đúng)

$\Rightarrow \sum \frac{1}{\sqrt{x^{4}+7x^{2}+1}}\geq \sum \frac{1}{x^{2}+x+1}\geq 1$




#513709 TOPIC: Xoay quanh $\sum \frac{1}{x^2+x+1}...

Đã gửi bởi Messi10597 on 18-07-2014 - 16:25 trong Bất đẳng thức và cực trị

Đề ngược dấu thì phải

Đặt : $x=\frac{bc}{a^{2}};y=\frac{ca}{b^{2}};z=\frac{ab}{c^{2}}$

$dpcm\Leftrightarrow \sum \frac{a^{4}}{a^{4}+a^{2}bc+b^{2}c^{2}}\geq 1$

Áp dụng Cauchy-Schwarz ta có

$\sum \frac{a^{4}}{a^{4}+a^{2}bc+b^{2}c^{2}}\geq \frac{(a^{2}+b^{2}+c^{2})^{2}}{(a^{4}+b^{4}+c^{4})+abc(a+b+c)+(a^{2}b^{2}+b^{2}c^{2}+c^{2}a^{2})}$

Mà $a^{2}b^{2}+b^{2}c^{2}+c^{2}a^{2}\geq abc(a+b+c)$

$\Rightarrow \sum \frac{a^{4}}{a^{4}+a^{2}bc+b^{2}c^{2}}\geq \frac{(a^{2}+b^{2}+c^{2})^{2}}{a^{4}+b^{4}+c^{4}+2(a^{2}b^{2}+b^{2}c^{2}+c^{2}a^{2})}=1$




#433938 [TSĐH 2013] Đề thi môn toán khối D

Đã gửi bởi Messi10597 on 09-07-2013 - 10:23 trong Thi TS ĐH

Bài 2:

$sin3x+cos2x-sinx=0$ $\Leftrightarrow cos2x+2cos2xsinx=0$

                                   $\Leftrightarrow cos2x(1+2sinx)=0$

                                   $\Leftrightarrow cos2x=0$

                                hoặc $sinx=-\frac{1}{2}$

                                   $\Leftrightarrow x=\frac{\pi }{4}+\frac{k\pi }{2}$

                                hoặc $x=-\frac{\pi }{6}+2k\pi$

                                hoặc $x=\frac{7\pi }{6}+2k\pi$

                                                $(k\epsilon \mathbb{Z})$




#433999 [TSĐH 2013] Đề thi môn toán khối D

Đã gửi bởi Messi10597 on 09-07-2013 - 14:52 trong Thi TS ĐH

Câu 5 (1,0 điểm). Cho hình chóp $S.ABCD$ có đáy là hình thoi cạnh $a$, cạnh bên $SA$ vuông góc với đáy, $\widehat{BAD}=120^o$, $M$ là trung điểm cạnh $BC$ và $\widehat{SMA}=45^o$. Tính theo $a$ thể tích của khối chóp $S.ABCD$ và khoảng cách từ điểm $D$ đến mặt phẳng $(SBC)$.

Diện tích đáy $S_{ABCD}=\frac{1}{2}AC.BD=\frac{1}{2}.a.a\sqrt{3}=a^{2}\frac{\sqrt{3}}{2}$

Nối A với M,S với M

Do $SA\perp mp(ABCD)\Rightarrow SA\perp AM$

Mà $\widehat{SMA}=45^{o}\Rightarrow \Delta SAM$ vuông cân tại A $\Rightarrow SA=AM$

Do $\widehat{BAD}=120^{o}\Rightarrow \widehat{ABC}=60^{o}$

Xét tam giác đều ABC cạnh a có AM là trung tuyến đồng thời là đường cao

Tính đc $AM=\frac{a\sqrt{3}}{2}$ $\Rightarrow SA=\frac{a\sqrt{3}}{2}$

Thể tích của khối chóp : $V_{S.ABCD}=\frac{1}{3}S_{ABCD}.SA=\frac{1}{3}.a^{2}\frac{\sqrt{3}}{2}.\frac{a\sqrt{3}}{2}=\frac{a^{3}}{4}$

Ta có: $S_{BCD}=\frac{1}{2}BC.DC.sin\widehat{BCD}=\frac{1}{2}a^{2}.sin120^{o}=\frac{a^{2}\sqrt{3}}{4}$

$\Rightarrow V_{S.BDC}=\frac{1}{3}.\frac{a\sqrt{3}}{2}.\frac{a^{2}\sqrt{3}}{4}=\frac{a^{3}}{8}$

Do $AM\perp BC;SA\perp mp(ABCD)\Rightarrow SM\perp BC$

$SM=AM\sqrt{2}=a\sqrt{\frac{3}{2}}$

$\Rightarrow S_{SBC}=\frac{1}{2}.SM.BC=\frac{1}{2}.a\sqrt{\frac{3}{2}}.a=a^{2}\frac{\sqrt{3}}{2\sqrt{2}}$

$\Rightarrow d(D;(SBC))=\frac{3.V_{S.BDC}}{S_{SBC}}=\frac{3.\frac{a^{3}}{8}}{a^{2}\frac{\sqrt{3}}{2\sqrt{2}}}=a\frac{\sqrt{3}}{2\sqrt{2}}$

Mọi nười xem có sai chỗ nào ko hộ em với ạ




#417717 $ (x-y)(x^2+y^2+xy+3)=3(x^2+y^2)+2 $

Đã gửi bởi Messi10597 on 10-05-2013 - 22:57 trong Phương trình - hệ phương trình - bất phương trình

Bạn ơi, ở đây tại sao chỉ lấy số a ở khoảng $-2\leq x< a\leqslant \frac{22}{3}$ mà không lấy khoảng khác bạn. Bạn chỉ cho mình cách lấy khoảng như vậy với.

À,đầu tiên tớ chọn  $a\epsilon \left [ -2;\frac{22}{3} \right ]$ là để cho a thỏa đk xác định của PT kia nhá,còn việc a>x hay a<x thì ko quan trọng đâu bạn ạ,quan trọng là chứng minh hàm f(x) đồng biến,nghĩa là x>a thì f(x) >f(a),hoặc x<a thì f(x)<f(a) thế thôi bạn ạ.




#420490 Chứng minh $a+b+c+\frac{1}{a}+\frac{1...

Đã gửi bởi Messi10597 on 23-05-2013 - 15:14 trong Bất đẳng thức và cực trị



thank các bạn. Mình có mấy bài nữa nè giúp mình nha.

 

Bài 3: Cho $x>0; y>0$. Tìm giá trị nhỏ nhất của biểu thức $P=5x+3y+\frac{12}{x}+\frac{16}{y}$.

Bài 4: Giải phương trình : $1+\sqrt[3]{x-16}=\sqrt[3]{x+3}$.

Bài 5: Cho hai số thực a,b thỏa mãn $a>b$ và $ab=2$. Tìm giá trị nhỏ nhất của biểu thức: $Q=\frac{a^{2}+b^{2}}{a-b}$.

Bài 4: Đặt $\sqrt[3]{x+3}=u ;\sqrt[3]{x-16}=v$

Ta có hệ $\left\{\begin{matrix} u-v=1 & & \\ u^{3}-v^{3}=19 & & \end{matrix}\right.$ $\Leftrightarrow \left\{\begin{matrix} u-v=1 & & \\ (u-v)^{3}+3uv(u-v)=19 & & \end{matrix}\right.$ $\Leftrightarrow \left\{\begin{matrix} u-v=1 & & \\ uv=6 & & \end{matrix}\right.$ 

Giải hệ trên ta đc: $(u;v)=(3;2);(-2;-3)$

Nếu $\left\{\begin{matrix} u=3 & & \\ v=2 & & \end{matrix}\right.\Leftrightarrow \left\{\begin{matrix} \sqrt[3]{x+3}=3 & & \\ \sqrt[3]{x-16}=2 & & \end{matrix}\right.\Leftrightarrow x=24$

Nếu $\left\{\begin{matrix} u=-2 & & \\ v=-3 & & \end{matrix}\right.\Leftrightarrow \left\{\begin{matrix} \sqrt[3]{x+3}=-2 & & \\ \sqrt[3]{x-16}=-3 & & \end{matrix}\right.\Leftrightarrow x=-11$

Xong rồi,còn bài hình như cậu viết thiếu đề đấy,cậu xem lại xem nhé




#434418 Tuyển Sinh Vào Lớp 10 THPT Chuyên Thăng Long 2013 - 2014

Đã gửi bởi Messi10597 on 10-07-2013 - 23:24 trong Tài liệu - Đề thi

Bài 1:

 $A=\sqrt{2+\sqrt{3}}.\sqrt{(2+\sqrt{2+\sqrt{3}})(2-\sqrt{2+\sqrt{3}})}$

     $=\sqrt{2+\sqrt{3}}.\sqrt{2-\sqrt{3}}$

     $=\sqrt{(2+\sqrt{3})(2-\sqrt{3})}$

     $=\sqrt{4-3}=1$




#417303 $ (x-y)(x^2+y^2+xy+3)=3(x^2+y^2)+2 $

Đã gửi bởi Messi10597 on 08-05-2013 - 18:49 trong Phương trình - hệ phương trình - bất phương trình

PT(1) suy ra y=x-2 thay vào PT (2) ta có 

$4\sqrt{x+2}+\sqrt{22-3x}=x^{2}+8$   $(-2\leq x\leq \frac{22}{3})$

$\Leftrightarrow 4(\sqrt{x+2}-2)+\sqrt{22-3x}-4=x^{2}-4$

$\Leftrightarrow \frac{4(x-2)}{\sqrt{x+2}+2}-\frac{3(x-2)}{\sqrt{22-3x}+4}=(x-2)(x+2)$

$\Leftrightarrow (x-2)(x+2+\frac{3}{\sqrt{22-3x}+4}-\frac{4}{\sqrt{x+2}+2})= 0$

TH1:x=2 thay vào (1) suy ra y=0

TH2: f(x)=$x+2+\frac{3}{\sqrt{22-3x}+4}-\frac{4}{\sqrt{x+2}+2}=0$ (*)

ta thấy x=-1 là 1 nghiệm của PT(*)

NHận xét rằng giả xử có số a thoả $-2\leq x< a\leq \frac{22}{3}$

ta có$\sqrt{x+2}< \sqrt{a+2} ;\sqrt{22-3x}> \sqrt{22-3a}$

suy ra $-\frac{4}{\sqrt{x+2}+2}< -\frac{4}{\sqrt{a+2}+2}$

           $\frac{3}{\sqrt{22-3x}+4}< \frac{3}{\sqrt{22-3a}+4}$

suy ra f(x)$<$ f(a) suy hàm f(x) đồng biến

suy x=-1 thì f(x)=0

       x<-1 thì f(x) <0

       x>-1 thì f(x)>0

suy ra x=-1 là nghiệm duy nhất của(*)

thay vào (1) ta có y=-3

đến đây bạn xem xem sai chỗ nào bảo tớ nhé